www.vorhilfe.de
Vorhilfe

Kostenlose Kommunikationsplattform für gegenseitige Hilfestellungen.
Hallo Gast!einloggen | registrieren ]
Startseite · Forum · Wissen · Kurse · Mitglieder · Team · Impressum
Forenbaum
^ Forenbaum
Status Englisch
  Status Grammatik
  Status Lektüre
  Status Korrekturlesen
  Status Übersetzung
  Status Sonstiges (Englisch)

Gezeigt werden alle Foren bis zur Tiefe 2

Navigation
 Startseite...
 Neuerdings beta neu
 Forum...
 vorwissen...
 vorkurse...
 Werkzeuge...
 Nachhilfevermittlung beta...
 Online-Spiele beta
 Suchen
 Verein...
 Impressum
Das Projekt
Server und Internetanbindung werden durch Spenden finanziert.
Organisiert wird das Projekt von unserem Koordinatorenteam.
Hunderte Mitglieder helfen ehrenamtlich in unseren moderierten Foren.
Anbieter der Seite ist der gemeinnützige Verein "Vorhilfe.de e.V.".
Partnerseiten
Weitere Fächer:

Open Source FunktionenplotterFunkyPlot: Kostenloser und quelloffener Funktionenplotter für Linux und andere Betriebssysteme
Forum "Folgen und Reihen" - Monotonie und Beschränktheit
Monotonie und Beschränktheit < Folgen und Reihen < eindimensional < reell < Analysis < Hochschule < Mathe < Vorhilfe
Ansicht: [ geschachtelt ] | ^ Forum "Folgen und Reihen"  | ^^ Alle Foren  | ^ Forenbaum  | Materialien

Monotonie und Beschränktheit: Schranken
Status: (Frage) beantwortet Status 
Datum: 12:43 Di 14.01.2014
Autor: gotoxy86

Ich hab die Folge [mm] a_n=\br{n^2}{2n-1}, [/mm] und davon will ich die Schranken finden, also ihre Beschränktheit finden.

Als erstes zeige ich, dass sie streng monoton wachsend ist:

[mm] a_{n+1}-a_n=\br{\left(n+1\right)^2}{2n+1}-\br{n^2}{2n-1}=\br{2n^2-1}{4n^2-1}>\br{1}{3}>0 [/mm]

In der Vorlesung wurde gesagt, dass ich jetzt aufgrund dieses Beweises schon sagen kann, dass 0 die untere Schranke ist.

Aber ich kann auch [mm] \br{a_{n+1}}{a_n}>1 [/mm] als Beweis für ihre Monotonie hernehmen, dann habe ich 1 als die untere Schranke.

Was ist denn jetzt die untere Schranke?

        
Bezug
Monotonie und Beschränktheit: Antwort
Status: (Antwort) fertig Status 
Datum: 13:07 Di 14.01.2014
Autor: fred97


> Ich hab die Folge [mm]a_n=\br{n^2}{2n-1},[/mm] und davon will ich
> die Schranken finden, also ihre Beschränktheit finden.
>  
> Als erstes zeige ich, dass sie streng monoton wachsend
> ist:
>  
> [mm]a_{n+1}-a_n=\br{\left(n+1\right)^2}{2n+1}-\br{n^2}{2n-1}=\br{2n^2-1}{4n^2-1}>\br{1}{3}>0[/mm]
>  
> In der Vorlesung wurde gesagt, dass ich jetzt aufgrund
> dieses Beweises schon sagen kann, dass 0 die untere
> Schranke ist.

?????

Es gibt nicht "die" untere Schranke ! gilt z.B. für eine Folge [mm] (c_n), [/mm] dass

    [mm] c_n \ge [/mm] 4711 für alle n

ist, so ist 4711 eine untere Schranke. Ebenso sind 2 und -123 untere Schranken von [mm] (c_n). [/mm]

Aus [mm] a_{n+1}-a_n [/mm] >0 folgt i.a. nicht, dass 0 eine untere Schranke ist.

Nimm z.B. [mm] $a_n [/mm] =- [mm] \bruch{1}{n}$ [/mm]

Bei Deiner obigen Folge [mm] a_n=\br{n^2}{2n-1} [/mm] sieht man doch durch genaues draufsehen, dass [mm] a_n [/mm] > 0 für alle n ist.


Aus [mm] a_{n+1}-a_n [/mm] > 0 für alle n folgt:

    [mm] a_n \ge a_1 [/mm]  für alle n.

Damit ist [mm] a_1 [/mm] die größte untere Schranke von [mm] (a_n) [/mm]


>  
> Aber ich kann auch [mm]\br{a_{n+1}}{a_n}>1[/mm] als Beweis für ihre
> Monotonie hernehmen,


> dann habe ich 1 als die untere
> Schranke.

Hä ? Wieso das denn ???  1 ist keine untere Schranke von [mm] (\br{n^2}{2n-1}) [/mm] !!!!

Edit: natürlich ist 1 eine untere Schranke

>  
> Was ist denn jetzt die untere Schranke?

"Die" untere Schranke gibt es nicht.

FRED


Bezug
                
Bezug
Monotonie und Beschränktheit: Lösung vom Lehrstuhl
Status: (Frage) beantwortet Status 
Datum: 13:38 Di 14.01.2014
Autor: gotoxy86

"Die Folge ist streng monoton wachsend, denn für alle [mm] n\inN [/mm] gilt [mm] 2n^2>1 [/mm] und daher

[mm] n^2(2n+1)=2n^3+n^2<2n^3+3n^2-1=(n^2+2n+1)(2n-1), [/mm]

also ist stets (beachte $2n+1>0$ und [mm] n^2>0) [/mm]

[mm] \br{a_{n+1}}{a_n}=\br{(n+1)^2}{2n+1}\br{2n-1}{n^2}>1, [/mm]

also ist stets [mm] a_n+1>a_n [/mm] (beachte [mm] a_n>0 [/mm] für alle [mm] n\inN). [/mm] Die Folge ist nicht nach oben beschränkt, denn für alle [mm] n\inN [/mm] gilt

[mm] a_n\ge\br{n^2}{2n}=\br{n}{2}. [/mm]

(Wegen Monotonie ist natürlich [mm] a_1=1 [/mm] eine untere Schranke der Folge.)
"

Ist das nun ein Fehler, oder was ist damit gemeint?

Bezug
                        
Bezug
Monotonie und Beschränktheit: Antwort
Status: (Antwort) fertig Status 
Datum: 13:41 Di 14.01.2014
Autor: fred97


> "Die Folge ist streng monoton wachsend, denn für alle
> [mm]n\inN[/mm] gilt [mm]2n^2>1[/mm] und daher
>
> [mm]n^2(2n+1)=2n^3+n^2<2n^3+3n^2-1=(n^2+2n+1)(2n-1),[/mm]
>
> also ist stets (beachte [mm]2n+1>0[/mm] und [mm]n^2>0)[/mm]
>
> [mm]\br{a_{n+1}}{a_n}=\br{(n+1)^2}{2n+1}\br{2n-1}{n^2}>1,[/mm]
>
> also ist stets [mm]a_n+1>a_n[/mm] (beachte [mm]a_n>0[/mm] für alle [mm]n\inN).[/mm]
> Die Folge ist nicht nach oben beschränkt, denn für alle
> [mm]n\inN[/mm] gilt
>
> [mm]a_n\ge\br{n^2}{2n}=\br{n}{2}.[/mm]
>
> (Wegen Monotonie ist natürlich [mm]a_1=1[/mm] eine untere Schranke
> der Folge.)"
>  
> Ist das nun ein Fehler, oder was ist damit gemeint?


Es wurde gezeigt, dass [mm] (a_n) [/mm] streng monoton wachsend ist und nicht nach oben beschränkt ist.

FRED

Bezug
                                
Bezug
Monotonie und Beschränktheit: Frage (beantwortet)
Status: (Frage) beantwortet Status 
Datum: 15:11 Di 14.01.2014
Autor: gotoxy86

Warum behauptet er einfach, dass es nach oben nicht berschänkt ist und nach unten mit 1 als Schranke?

Bezug
                                        
Bezug
Monotonie und Beschränktheit: Antwort
Status: (Antwort) fertig Status 
Datum: 15:13 Di 14.01.2014
Autor: Gonozal_IX

Hiho,

> Warum behauptet er einfach, dass es nach oben nicht berschänkt ist und nach unten mit 1 als Schranke?

weil beides gezeigt wurde!
Guckst du dir die Beweise überhaupt an oder nur die Schlußsätze?

Es wurde gezeigt:

[mm] $a_n$ [/mm] ist monoton wachsend [mm] $\Rightarrow a_n \ge a_1 [/mm] = 1$

Also ist es nach unten durch 1 beschränkt.

Dann wurde gezeigt:

[mm] $a_n \ge \bruch{n}{2}$ [/mm] und [mm] \bruch{n}{2} [/mm] ist offensichtlich nicht nach oben  beschränkt und damit erst recht nicht [mm] a_n [/mm]

Man man man
Gono.



Bezug
                                                
Bezug
Monotonie und Beschränktheit: Frage (beantwortet)
Status: (Frage) beantwortet Status 
Datum: 16:50 Di 14.01.2014
Autor: gotoxy86

Es wurde doch gerade gesagt, dass man mit Monotonie nicht die Beschränktheit beweisen kann.

Bezug
                                                        
Bezug
Monotonie und Beschränktheit: Antwort
Status: (Antwort) fertig Status 
Datum: 17:06 Di 14.01.2014
Autor: schachuzipus

Hallo,

> Es wurde doch gerade gesagt, dass man mit Monotonie nicht
> die Beschränktheit beweisen kann.

Da die Folge monoton wachsend ist (und sämtliche Gleider positiv sind), ist das erste Glied das kleinste, also haben wir Beschränktheit nach unten.

Weiter ist die Folge UNbeschränkt nach oben.

Wo genau ist dein Problem??

Gruß

schachuzipus

Bezug
                        
Bezug
Monotonie und Beschränktheit: Antwort
Status: (Antwort) fertig Status 
Datum: 13:59 Di 14.01.2014
Autor: Gonozal_IX

Hiho,

> (Wegen Monotonie ist natürlich [mm]a_1=1[/mm] eine untere Schranke der Folge.)"

ob 1 eine untere Schranke ist, hängt davon ab, wie bei euch die natürlichen Zahlen definiert sind.

Im Fall von [mm] $\IN [/mm] = [mm] \{0,1,2,3\ldots\}$ [/mm] ist 1 keine untere Schranke, im Fall von [mm] $\IN [/mm] = [mm] \{1,2,3\ldots\}$ [/mm] schon.

Gruß,
Gono.

Bezug
                                
Bezug
Monotonie und Beschränktheit: Frage (beantwortet)
Status: (Frage) beantwortet Status 
Datum: 14:39 Di 14.01.2014
Autor: gotoxy86

Darf [mm] a_n [/mm] reell sein aber n nicht?

Oder dürfen beide nicht reell sein?

Bezug
                                        
Bezug
Monotonie und Beschränktheit: Antwort
Status: (Antwort) fertig Status 
Datum: 14:55 Di 14.01.2014
Autor: fred97


> Darf [mm]a_n[/mm] reell sein aber n nicht?


>  
> Oder dürfen beide nicht reell sein?

Du hast [mm] a_n=\br{n^2}{2n-1} [/mm] mit n [mm] \in \IN [/mm] ( oder n [mm] \in \IN_0) [/mm]

Es ist also n eine natürliche Zahl, also auch eine reelle, und [mm] a_n \in \IR. [/mm]

FRED


Bezug
Ansicht: [ geschachtelt ] | ^ Forum "Folgen und Reihen"  | ^^ Alle Foren  | ^ Forenbaum  | Materialien


^ Seitenanfang ^
www.englischraum.de
[ Startseite | Forum | Wissen | Kurse | Mitglieder | Team | Impressum ]